LSAT and Law School Admissions Forum

Get expert LSAT preparation and law school admissions advice from PowerScore Test Preparation.

 Administrator
PowerScore Staff
  • PowerScore Staff
  • Posts: 8917
  • Joined: Feb 02, 2011
|
#23827
Complete Question Explanation

Resolve the Paradox. The correct answer choice is (E)

The author of this economics-based passage is presents a discussion of consumer behavior during periods of inflation. At first, consumers tend to spend more in an effort to save on the future increased prices. Then, however, they tend to put off routine purchases. The correct answer choice will provide some explanation of why consumers begin to put off such routine purchases despite the fact that there exists the same expectation of rising prices, and despite the fact that salaries increase in an inflationary economy as well.

Correct answer choice (E) provides some insight: regardless of increasing salaries, the average consumer’s purchasing power does not keep up in an inflationary economy, which would explain why we might not see the continued increased spending as we might otherwise expect.
 prep88
  • Posts: 37
  • Joined: Jan 20, 2015
|
#18590
Hello,

Answer choice (E) does make sense, but I don't get why (D) is wrong. If consumers know that when they refrain from purchasing for some time then prices will fall, that would explain why they are holding back from making those expenditures, right?

Thanks!
 Andrew Ash
PowerScore Staff
  • PowerScore Staff
  • Posts: 32
  • Joined: Sep 15, 2014
|
#18606
Hi Prep,

Great to hear from you again!

I'm going to be a little picky with the actual wording of answer choice (D) here versus how you interpreted it. The answer choice itself says: "If significant numbers of consumers are unable to make purchases, prices will eventually fall but salaries will not be directly affected." That has two small, but important, differences from the way you've paraphrased the answer in your post:

a) The answer choice talks about something that will happen, not about consumers knowing that it will happen.

b) The answer choice talks about consumers who are "unable" to make purchases, not about consumers who are intentionally "holding back" from making purchases.

So while I agree that consumers intentionally delaying purchases in order to encourage deflation would resolve the paradox, as you describe, I don't think that's what answer choice (D) actually said.

I think the larger point here is to make sure that you're not doing work to make an answer choice correct. Those two jumps that you made were small - tiny, in fact - but they made answer choice (D) more attractive, which made this question more difficult. Remember, get mean with your answer choices. Approach them skeptically, try to think about how they couldn't possibly correct as you're reading them, and you're much less likely to be tempted by answer choices that almost get the job done, but aren't quite good enough.

I hope this helps!

Thanks,
Andrew
 lday4
  • Posts: 44
  • Joined: May 05, 2016
|
#25081
Can you explain why D is incorrect? Thanks!
User avatar
 Stephanie Oswalt
PowerScore Staff
  • PowerScore Staff
  • Posts: 811
  • Joined: Jan 11, 2016
|
#25109
Hi lday4,

Thanks for your question. Generally speaking, we need a bit more input from you before we delve into a discussion of a particular LR question. Ultimately, it won't be us who are taking the test; it's you! :-) Our goal is to help you cultivate the analytical ability to approach these questions on your own, which is why you need to help us help you first.

Here's what I'd like you to do:

1. Describe your approach to the stimulus. Did you understand the argument, if any, from a structural standpoint? What is the conclusion, and what evidence is the author using in support of that conclusion?

2. Did you prephrase an answer to the question in the stem? If so, what was your prephrase?

3. What exactly made the answer choices you have listed particularly attractive? Did you use any question type-specific test (e.g. Assumption Negation Technique)?

Thanks,
Stephanie
 biskam
  • Posts: 124
  • Joined: Aug 18, 2017
|
#40331
I chose E but was tempted by A... is A incorrect because it's kind of just a nice fact but it doesn't explain why ppl put off routine purchases whereas E gives a better reason why?

Thanks!
 Adam Tyson
PowerScore Staff
  • PowerScore Staff
  • Posts: 5153
  • Joined: Apr 14, 2011
|
#40344
Exactly right, biskam - it's a "that's nice - so what?" answer choice. The paradox here is that when inflation begins, spending increases, but as inflation lasts, spending drops even though the same motivation that spurred the initial spending increase remains in place. Why is the motivation to spend at first good enough, but later it isn't? Answer A tells us nothing about why that shift takes place (and in fact might be at some odds with that shift since spending is stated to initially increase), while answer E, the correct answer, does.
 yusrak
  • Posts: 22
  • Joined: Mar 19, 2020
|
#75131
Hi Powerscore,

I was stuck between D and E as well. I read the explanation above and I thought I understood but then a few questions arose.

I reasoned that choice E is correct because regardless of increasing salaries, the rate at which they increase is not the same as the rate at which the prices increase. Thus, the average consumer’s purchasing power does not keep up in an inflationary economy. This is why continued increased spending does not occur during inflation.

I reasoned that choice D is incorrect because it addresses a different group of people (unable to make purchases) than the group identified in the stimulus (consumers postponing). But can I infer that the group in the stimulus is intentionally postponing? Also, couldn’t the group indicated in choice D be re-categorized as “unwilling to make purchases?"

I thought answer D was correct because it solves not only the paradox but also the situation; if consumers know that delaying purchases will lower prices, then they will eventually be able to make purchases again in the future. But in answer choice E, if they know their purchasing power decreases even for routine goods, how are they going to continue buying essential goods? Perhaps I am over analyzing this by considering the future consequences of their actions when I don't have to be concerned about that.

Also, generally speaking, do all strengthen questions have to follow the same tense? So, for example, if the stimulus is in present tense, as in this question, the answer must be in present tense (like choice E) and it cannot be in future tense (like choice D).
 Adam Tyson
PowerScore Staff
  • PowerScore Staff
  • Posts: 5153
  • Joined: Apr 14, 2011
|
#75950
This is actually a Resolve the Paradox question, Yusra, and not a Strengthen. Resolve answers will generally give us a cause for whatever unusual thing happened in the stimulus. So here, we want to know what causes people to hold back from making purchases during protracted periods inflation, when at the beginning of the period the are buying more than usual. Look for what causes a change in their behavior over time.

Answer D does nothing to explain the cause of their behavior, but only tells us about a possible effect, and even then it is only an effect that happens after people no longer have a choice. Being unable is not being unwilling - "can't" is very different from "won't." To suggest that maybe people are holding back in order to make prices come down is to stretch that answer beyond recognition. We can't help the answer choices like that, but need to take them at face value. Answer E gives us a cause for the change - people can't afford to keep it up, so they stop buying. That answer needs no help to resolve this paradox!

Get the most out of your LSAT Prep Plus subscription.

Analyze and track your performance with our Testing and Analytics Package.